Evaluate -6 - (-3)​

Answers

Answer 1
Answer:

Answer: -3

Step-by-step explanation: In this problem we are asked to subtract -6 - (-3). It's important to understand that when we have minus a negative, we can change it to plus a positive.

Now we have the following.

-6 + 3

Starting at 0 on our number line, -6 moves us 6 units to the left. Then from there, positive 3 moves us 3 units back to the right and we end up at -3.

Therefore, -6 - (-3) = -3


Related Questions

If two sides of a triangle are 10 units and 13 units,a possible third side for the could be?
a window is shaped like a parallelogram the base of the window is 27 inches its area is 445.5 square inches. what is the height of the window?
Bill measures the circumference of a circular fish pond to be 28 ft. What is the diameter of the fish pond, rounded to the nearest foot? Use 3.14 to approximate pi.
When solving the equation x2-8x-7=0 by completing the square
Is - 3/5 greater, less than or equal to - 4/5

25% of what number is 9

Answers

The solution for the number is 36.

Used the concept of the percentage that states,

To Calculate the percent of a number, divide the number by the whole number and multiply by 100.

Let us assume that a number x that satisfies 25% of x is 9.

Hence it can be written as,

25\% \  \text {of }x  = 9

Solve for x,

(25)/(100) * x = 9

25x = 900

x = 36

Therefore, the solution is 36.

To learn more about percentages visit:

brainly.com/question/24877689

#SPJ6

Answer: 36

Step-by-step explanation:

Let the number be x

Thus we can write:

25\%* x=9

(25)/(100)* x=9

x=(9* 100)/(25)

x=36

Thus 25% of 36 is 9.

In the diagram, find the measure of Angle y and Angle x.

PLEASE HELP ME!!

Answers

Answer:

Part 1) The measure of angle x is 24°

Part 2) The measure of angle y is 66°

Step-by-step explanation:

step 1

Find the measure of angle m∠IKL

we know that

m∠JKI+m∠IKL=180° ----> supplementary angles (form a linear pair)

we have

m∠JKI=48°

substitute

48°+m∠IKL=180°

m∠IKL=180°-48°=132°

step 2

Find the measure of angle x

we know that

The triangle IKL is an isosceles triangle

so

m∠KIL=m∠KLI=x

Remember that

The sum of the interior angles of a triangle must be equal to 180 degrees

m∠IKL+2x=180°

132°+2x=180°

2x=180°-132°

x=24°

step 3

In the right triangle JIL

Angles x and y are complementary

so

x+y=90°

24°+y=90°

y=90°-24°

y=66°

Kevin claims he can draw a trapezoid with three right angles. Is this possible? Explain.

Answers

no, if you were to have three right angles, then the fourth angle would also have to be a right angle.  This would make the shape either a square or a rectangle, not a trapezoid.
No, because a trapezoid is defined as a quadrilateral with EXACTLY one pair of parallel sides (the other two cannot be parallel). Because the sum of the angles in a quadrilateral sum up to 360°, and three of the angles are each 90°, that means the fourth angle must equal (360 - 3(90)) = 90°, so therefore this figure would be a rectangle, not a trapezoid.

wade wants to buy sweaters. He has $175 and each sweater costs $12.00. Write and solve an inequality to find how many sweaters he can buy and still have at least $55.

Answers

Wade wants to have at least $55 left, so the sign will be ≤ (greater than or equal to 55) One sweater costs $12. You can use x to represent an unkown number of sweaters, so $12x. You are subtracting 12x by your budget of $175. When put together, the inequality is $175-$12x ≤ $55.Now to solve. You want to get x alone so subtract 175. You must do this to both sides. Now you have -12x≤ -120. Divide both sides by -12 and your final answer is -120/-12 ≥ 10. (Notice the sign changes when you divide by a negative)So Wade cannot buy more than 10 sweaters to have $55 left over. Phoo that was a workout. It can be difficult but once you get the hang of it it comes easy. If you need more help just ask :D
X is the number of sweaters Wade buys.
175$-12x≥55$.
Solve the equation to get X = 10 sweaters.

HELP PLEASE I BEG YOU IR EALLY NEED HELP!!!!/

Answers

925 in= C
question 7) A
question 8) A
question 9) a
question 1) B
question 2)D
question 3) A
question 4)D
question5) C

the answer is 1.b        2. d              3. c            4. c

Your teacher buys a 5 pound bad of Smarties, for $12.50. What is the unit rate for Smarties.

Answers

so 5 pounds=12.5
divide 12.5 by 5 to get dollars per pound
$2.5 per pouind
You have to divide 12.50÷5
12.50÷5=2.5
The Unit rate for Smarties is $2.50 per pound.